peg_city
Thanks Received: 3
Forum Guests
 
Posts: 152
Joined: January 31st, 2011
Location: Winnipeg
 
 
trophy
First Responder
 

Q23 - In a certain municipality, a

by peg_city Sun Mar 20, 2011 4:26 pm

Can someone break the question and answers for me?

I don't even know where to start on this one.
 
giladedelman
Thanks Received: 833
LSAT Geek
 
Posts: 619
Joined: April 04th, 2010
 
This post thanked 2 times.
 
 

Re: Q23 - In a certain municipality, a

by giladedelman Tue Mar 22, 2011 5:46 pm

Oooh boy, this is a tricky one! Lots going on here.

Let's start with the conclusion: the evidence in the case was inadmissible. Straightforward enough. But how does the judge get there? We have a series of premises:

- only cause for police chase was suspect's flight
- flight alone does not create reasonable suspicion of criminal act
- evidence collected in an illegal chase is inadmissible

So, let's think about this one. The judge concludes that the evidence is inadmissible. According to the premises, the only way we can decide that evidence is inadmissible is if it's collected in an illegal chase. But where does the argument say that the chase was illegal?

Aha! It doesn't say the chase was illegal! So we need a principle that connects what we do know about the chase -- the only cause was the suspect's flight, which alone does not create reasonable suspicion of a criminal act -- to what we want to know, which is that it was illegal.

That's why (C) is correct. If we adopt the principle that police can legally chase someone only when the person's actions have created reasonable suspicion of a criminal act, then we can say that this particular chase was indeed illegal, because all the guy did was run away. If the chase was illegal, then the judge's conclusion that the evidence is inadmissible follows logically.

(A) is incorrect because one of the premises is that the police gave chase only because the suspect fled; there were no other significant factors.

(B) doesn't help us because the argument doesn't have anything to do with when people can legally flee; it has to do with when the police can legally chase you.

(D) might be tempting, but it only really pertains to a premise; we already know that flight from the police in itself doesn't create reasonable suspicion of a criminal act. Whether the flight itself should be considered a criminal act is not at issue.

(E) is also tempting, but it's actually the negated version of what we want. We need to say that having no suspicion of a criminal act means the police can't give chase legally; we don't care about what happens when you do have suspicion, since in this case there was none.

Does that clear this one up for you?
User avatar
 
WaltGrace1983
Thanks Received: 207
Atticus Finch
Atticus Finch
 
Posts: 837
Joined: March 30th, 2013
 
 
trophy
Most Thanked
trophy
Most Thankful
trophy
First Responder
 

Re: Q23 - In a certain municipality, a

by WaltGrace1983 Tue Feb 11, 2014 3:22 pm

This one was a tricky one for me too and so I thought I'd add some analysis here. This question is tricky because we are so used to connecting the premise to the final conclusion. In this case, we are really just connecting a premise to an intermediate conclusion. Let's dive into what this argument is saying:

(1) Only cause for police chase was suspect's flight
(2) Flight, by itself, does not create a reasonable suspicion
(3) Evidence collected during an illegal chase is inadmissible

(C) Evidence in this case was inadmissible

The jump from (3) to (C) seems fairly rock solid because we assume from the premises that this case is illegal. I think this is perfectly fine to assume. However, what is not perfectly fine is assuming that (1) → (2) → (3) → (C) in a seamless manner. There is a gap between ~Reasonable Suspicion and ~Legal. This gap is shown between (2) and (3). So in order to justify the judge's conclusion, we absolutely have to address that gap. Remember, when a principle is given to justify, it is basically a sufficient assumption question. Let's look at the answer choices, looking for something that says "~Reasonable Suspicion → ~Legal Chase" or "Legal Chase → Reasonable Suspicion."

(A) No other significant factors involved → No reasonable suspicion

What do we know? We know that no other significant factors are involved ("the only cause for the police gave chase was..."). So we satisfied the sufficient condition. Thus, we can conclude from this answer choice that there was no reasonable suspicion. Okay, awesome, but what does that do?! Nothing! It tells us what we already know. This is essentially a premise booster.

(B) Involved in a criminal act → ~Legally flee

This one looks fairly good. We do know that the suspect was involved in a criminal act ("the suspect...discarded the illegal weapon after the police gave chase"). Thus, the sufficient condition is satisfied and this leads us to the idea that the suspect could not legally flee. This really seems to justify the conclusion now so whats the problem? The problem is that we are talking about fleeing, not chasing. The whole argument is about, "evidence collected during an illegal chase is inadmissible." Another thing too about this one is, even if you didn't see that flee doesn't equal chase, you should be apprehensive that this doesn't really address the rest of the premises and/or the main gap in the argument.

(D) Flight from the police → ~Criminal Act.

This is addressing something that is basically out of scope. We are talking about in this argument a judge overturning a suspect's conviction of an illegal weapon. Maybe we could take (D) as true. It doesn't justify this court case though. Maybe, if (D) is true, then the suspect did commit a crime. But so what? It was a different crime; a crime we are not concerned with. Out of scope.

(E) Reasonable Suspicion → Legal Chase

Ohhh so close! However, here is a good opportunity for an important lesson in logic. If our argument gives us ________ → ~Legal Chase then we must always have something that matches that. This says _________ → Legal Chase. This can never be right. This is a reversal of what we want.

(C) Legally Chase → Reasonable Suspicion

Tricky LSAT! It gave us the contrapositive of what we want. We know better. Let's turn this into "~Reasonable Suspicion → ~Legally Chase." From the stimulus, the sufficient condition is satisfied and this gives us the necessary condition, the ~Legally chase. Look how beautiful this matches up...

(1) F
(2) F → ~RS
(C) ~RS → ~L
(3) ~L → ~A

~A

Perfect! Now once again, why is this question tricky? (A) is a tempting premise booster, (B) makes a very subtle scope shift, (D) gives us a tempting out of scope answer, and (E) is a reversal of what we want. This is one of the more challenging set of answer choices to sift through, in my opinion, if you don't have a very rock solid understanding of the gap.
 
AlexM563
Thanks Received: 1
Vinny Gambini
Vinny Gambini
 
Posts: 6
Joined: June 24th, 2018
 
This post thanked 1 time.
 
 

Re: Q23 - In a certain municipality, a

by AlexM563 Tue Jul 24, 2018 9:25 pm

I wanted to add to this discussion that I misread the question stem as “the judges view conforms best to which of the following principles”, in which case A would be correct. So A is also a trap answer for students who fail to pay enough attention to the question stem.